Ανισότητα με τρείς μεταβλητές

Συντονιστές: achilleas, emouroukos, silouan

gschwindi
Δημοσιεύσεις: 16
Εγγραφή: Δευ Μαρ 11, 2019 6:23 pm

Ανισότητα με τρείς μεταβλητές

#1

Μη αναγνωσμένη δημοσίευση από gschwindi » Δευ Μαρ 11, 2019 6:40 pm

Καλησπέρα, το παρακάτω αποτελεί την πρώτη μου δημοσίευση στο mathematica.
Είναι ένα θέμα εμπνευσμένο από μια άσκηση του βιβλίου "Μαθηματικοί Διαγωνισμοί 1" Χαράλαμπου Στεργίου και Σιλουανού Μπραζιτίκου.
Έχω βρεί μια λύση εγώ αλλα θα ήθελα και άλλες ιδέες.

Έστω a,b,c θετικοί πραγματικοί αριθμοί. Να αποδείχθεί πως:

a^{3} + b^{3} + c^{3} \geq \sqrt{\frac{abc}{2}}(a\sqrt{b+c}+b\sqrt{c+a}+c\sqrt{a+b}).


Πότε ισχύει η ισότητα;


(Ζητώ συγνώμη για αυτή την άβολη χρήση LaTex.)



Λέξεις Κλειδιά:
Άβαταρ μέλους
matha
Γενικός Συντονιστής
Δημοσιεύσεις: 6423
Εγγραφή: Παρ Μάιος 21, 2010 7:40 pm
Τοποθεσία: Θεσσαλονίκη

Re: Ανισότητα με τρείς μεταβλητές

#2

Μη αναγνωσμένη δημοσίευση από matha » Δευ Μαρ 11, 2019 6:58 pm

Μια απόδειξη, αλλά είμαι σίγουρος ότι υπάρχουν και άλλες:

Λόγω ομογένειας μπορούμε να υποθέσουμε ότι \displaystyle{abc=1,} Οπότε έχουμε να αποδείξουμε ότι

\displaystyle{a^3+b^3+c^3\geq \sum a\sqrt{\frac{b+c}{2}}.}

Από την ΑΜ-ΓΜ είναι

\displaystyle{\frac{1}{2}\left(a^2+\frac{b+c}{2}\right)\geq a\sqrt{\frac{b+c}{2}}},

οπότε αρκεί να αποδειχθεί ότι

\displaystyle{2(a^3+b^3+c^3)\geq a^2+b^2+c^2+a+b+c.}

Αυτή είναι συνέπεια των \displaystyle{a^3+b^3+c^3\geq a^2+b^2+c^2} και \displaystyle{a^3+b^3+c^3\geq a+b+c.}

Για την πρώτη:

Από την ανισότητα των δυνάμεων έχουμε

\displaystyle{3(a^3+b^3+c^3)^2\geq (a^2+b^2+c^2)^3,}

άρα

\displaystyle{(a^3+b^3+c^3)^2\geq (a^2+b^2+c^2)^2\left(\frac{a^2+b^2+c^2}{3}\right)\geq (a^2+b^2+c^2)^2,}

αφού από ΑΜ-ΓΜ είναι \displaystyle{a^2+b^2+c^2\geq 3.}

Για τη δεύτερη εργαζόμαστε αναλόγως, βασιζόμενοι στην \displaystyle{9(a^3+b^3+c^3)\geq (a+b+c)^3.}


Μάγκος Θάνος
Άβαταρ μέλους
grigkost
Διαχειριστής
Δημοσιεύσεις: 3053
Εγγραφή: Πέμ Δεκ 18, 2008 12:54 pm
Τοποθεσία: Ιωάννινα
Επικοινωνία:

Re: Ανισότητα με τρείς μεταβλητές

#3

Μη αναγνωσμένη δημοσίευση από grigkost » Δευ Μαρ 11, 2019 7:11 pm

καλώς όρισες στο mathematica.gr

και, παρεμπιπτόντως,
gschwindi έγραψε:
Δευ Μαρ 11, 2019 6:40 pm
...(Ζητώ συγνώμη για αυτή την άβολη χρήση LaTex.)
η γραφή του τύπου είναι άψογη.


{\color{dred}\Gamma\!\rho\,{\rm{H}}\gamma\varnothing\varrho{\mathscr{H}}\varsigma \ {\mathbb{K}}\,\Omega\sum{\rm{t}}{\mathscr{A}}\,{\mathbb{K}}\!\odot\varsigma
Prødigy

Re: Ανισότητα με τρείς μεταβλητές

#4

Μη αναγνωσμένη δημοσίευση από Prødigy » Δευ Μαρ 11, 2019 7:16 pm

Έχουμε ότι a^3+b^3+c^3\geq ab(a+b)+c^3\geq 2c\sqrt{abc(a+b)}

Κάνοντας το ίδιο στα άλλα μέλη προκύπτει a^3+c^3+b^3\geq 2b\sqrt{abc(a+c)} και b^3+c^3+a^3\geq 2a\sqrt{abc(b+c)}

Αρκεί να αποδείξουμε ότι 2a\sqrt{abc(b+c)}\geq \frac{a\sqrt{abc(b+c)}}{\sqrt{2}}\Leftrightarrow 8> 1

Όμοια δουλεύουμε και στα υπόλοιπα μέλη και προκύπτει το ζητούμενο.

Edit:Η λύση είναι λανθασμένη
τελευταία επεξεργασία από Prødigy σε Δευ Μαρ 11, 2019 11:20 pm, έχει επεξεργασθεί 5 φορές συνολικά.


gschwindi
Δημοσιεύσεις: 16
Εγγραφή: Δευ Μαρ 11, 2019 6:23 pm

Re: Ανισότητα με τρείς μεταβλητές

#5

Μη αναγνωσμένη δημοσίευση από gschwindi » Δευ Μαρ 11, 2019 7:32 pm

Prødigy έγραψε:
Δευ Μαρ 11, 2019 7:16 pm
Έχουμε ότι a^3+b^3+c^3\geq ab(a+b)+c^3\geq 2c\sqrt{abc(a+b)}

Κάνοντας το ίδιο στα άλλα μέλη προκυπτει 2b\sqrt{abc(a+c)} και 2c\sqrt{abc(c+a)}

Αρκεί να αποδείξουμε ότι 2a\sqrt{abc(b+c)}\geq \frac{a\sqrt{abc(a+b)}}{2}\Leftrightarrow 8> 1

Όμοια δουλεύουμε και στα υπόλοιπα μέλη και προκύπτει το ζητούμενο.
Έτσι όπως κοιτάω την πρότασή σου δεν βγαίνει, για κοίτα την ξανά.


Άβαταρ μέλους
Ορέστης Λιγνός
Δημοσιεύσεις: 1835
Εγγραφή: Κυρ Μάιος 08, 2016 7:19 pm
Τοποθεσία: Χαλάνδρι Αττικής
Επικοινωνία:

Re: Ανισότητα με τρείς μεταβλητές

#6

Μη αναγνωσμένη δημοσίευση από Ορέστης Λιγνός » Δευ Μαρ 11, 2019 11:39 pm

gschwindi έγραψε:
Δευ Μαρ 11, 2019 6:40 pm
Καλησπέρα, το παρακάτω αποτελεί την πρώτη μου δημοσίευση στο mathematica.
Είναι ένα θέμα εμπνευσμένο από μια άσκηση του βιβλίου "Μαθηματικοί Διαγωνισμοί 1" Χαράλαμπου Στεργίου και Σιλουανού Μπραζιτίκου.
Έχω βρεί μια λύση εγώ αλλα θα ήθελα και άλλες ιδέες.

Έστω a,b,c θετικοί πραγματικοί αριθμοί. Να αποδείχθεί πως:

a^{3} + b^{3} + c^{3} \geq \sqrt{\frac{abc}{2}}(a\sqrt{b+c}+b\sqrt{c+a}+c\sqrt{a+b}).


Πότε ισχύει η ισότητα;


(Ζητώ συγνώμη για αυτή την άβολη χρήση LaTex.)
Καλησπέρα.

Από Cauchy-Schwarz προκύπτει \displaystyle \sum a\sqrt{b+c} \leqslant \sqrt{\sum a^2 \cdot \sum (\sqrt{b+c})^2}, οπότε αρκεί \displaystyle (\sum a^3)^2 \geqslant \sum a^2 \sum a \cdot abc.

Όμως, από Cheybychev (ή Cauchy-Schwarz) προκύπτει \displaystyle \sum a^2 \sum a \leqslant 3\sum a^3, οπότε αρκεί \displaystyle (\sum a^3)^2 \geqslant 3\sum a^3 \cdot abc \Rightarrow \sum a^3 \geqslant 3abc, που είναι η ΑM-GM.


Κερδίζουμε ό,τι τολμούμε!
Άβαταρ μέλους
matha
Γενικός Συντονιστής
Δημοσιεύσεις: 6423
Εγγραφή: Παρ Μάιος 21, 2010 7:40 pm
Τοποθεσία: Θεσσαλονίκη

Re: Ανισότητα με τρείς μεταβλητές

#7

Μη αναγνωσμένη δημοσίευση από matha » Τρί Μαρ 12, 2019 12:09 am

matha έγραψε:
Δευ Μαρ 11, 2019 6:58 pm
...
οπότε αρκεί να αποδειχθεί ότι

\displaystyle{2(a^3+b^3+c^3)\geq a^2+b^2+c^2+a+b+c.}
Και ένα διαφορετικό τελείωμα:

Έχουμε να αποδείξουμε ότι

\displaystyle{(2a^3-a^2-a)+(2b^3-b^2-b)+(2c^3-c^2-c)\geq 0}

όταν \displaystyle{a,b,c>0, abc=1.}

Αυτό είναι συνέπεια της ανισότητας

\displaystyle{2x^3-x^2-x\geq 3\ln x~~\forall x>0,}

η οποία π.χ. αποδεικνύεται με τη βοήθεια της \displaystyle{x-1\geq \ln x. }


Μάγκος Θάνος
gschwindi
Δημοσιεύσεις: 16
Εγγραφή: Δευ Μαρ 11, 2019 6:23 pm

Re: Ανισότητα με τρείς μεταβλητές

#8

Μη αναγνωσμένη δημοσίευση από gschwindi » Τρί Μαρ 12, 2019 12:28 am

Βάζω και εγώ την μία από τις δύο λύσεις μου, για να αφήσω και άλλους να βρουν την άλλη. Παρατηρούμε πως
\sqrt{\frac{a^{3}bc(b+c)}{2}}\leq\frac{2a^{3}+bc(b+c)}{4} \leq \frac{2a^{3}+b^{3}+c^{3}}{4}, με ισότητα και στα τρία μέλη αν και μόνο αν  a=b=c.
Εφαρμόζοντας κυκλικά το ίδιο και αθροίζοντας, παίρνουμε την προς απόδειξη.


Άβαταρ μέλους
Demetres
Γενικός Συντονιστής
Δημοσιεύσεις: 8989
Εγγραφή: Δευ Ιαν 19, 2009 5:16 pm
Τοποθεσία: Λεμεσός/Πύλα
Επικοινωνία:

Re: Ανισότητα με τρείς μεταβλητές

#9

Μη αναγνωσμένη δημοσίευση από Demetres » Τρί Μαρ 12, 2019 3:50 pm

Λίγο διαφορετικά από τον Ορέστη:

Από Cauchy-Schwarz (ή ανισότητα δυνάμεων) και ΑΜ-ΓΜ έχουμε:

\displaystyle  \sum{a\sqrt{b+c} \leqslant \sqrt{3}\sqrt{\sum \left(a^2b+a^2c\right)} \leqslant \sqrt{3}\sqrt{\sum \left(\frac{2a^3+b^3}{3}+\frac{2a^3+c^3}{3}\right)} = \sqrt{3(a^3+b^3+c^3)}

Πάλι από ΑΜ-ΓΜ έχουμε και \displaystyle  \sqrt{abc} \leqslant \sqrt{\frac{a^3+b^3+c^2}{3}}

Πολλαπλασιάζοντας παίρνουμε το ζητούμενο.


gschwindi
Δημοσιεύσεις: 16
Εγγραφή: Δευ Μαρ 11, 2019 6:23 pm

Re: Ανισότητα με τρείς μεταβλητές

#10

Μη αναγνωσμένη δημοσίευση από gschwindi » Τρί Μαρ 12, 2019 4:33 pm

Ας το δούμε και αλλιώς.
Θα μπορούσε κάποιος να προσεγγίσει το θέμα και ως εξής:

Απο C.S είναι (a^3+b^3+c^3)(\frac{b+c}{a}+\frac{c+a}{b}+\frac{a+b}{c})\geq (a\sqrt{b+c}+b\sqrt{c+a}+c\sqrt{a+b})^2. Ισότητα όταν \sqrt{\frac{a^3}{\frac{b+c}{a}}} = \sqrt{\frac{b^3}{\frac{c+a}{b}}} = \sqrt{\frac{c^3}{\frac{a+b}{c}}}.

Θέτουμε abc = k, k \in \mathbb{R}, k > 0.

Τότε, \frac{b+c}{a} = \frac{bc(b+c)}{k}\leq \frac{b^3+c^3}{k}, ισότητα όταν  b = c .

Κάνοντας το ίδιο και αθροίζοντας τα τρία κλάσματα παίρνουμε ότι,

\frac{2}{abc}(a^3+b^3+c^3)^2 \geq (a\sqrt{b+c}+b\sqrt{c+a}+c\sqrt{a+b})^2..

Απο όπου το ζητούμενο έπεται.

Για να ισχύει η ισότητα, μεταξύ των δύο άκρων πρέπει να ισχύει η ισότητα μεταξύ του μέσου και των δυο άκρων. Αυτή επιτυγχάνεται όταν a=b=c.


Απάντηση

Επιστροφή σε “Άλγεβρα - Επίπεδο Θαλή/Ευκλείδη (Seniors)”

Μέλη σε σύνδεση

Μέλη σε αυτήν τη Δ. Συζήτηση: Δεν υπάρχουν εγγεγραμμένα μέλη και 8 επισκέπτες